матриц – Решатель линейной системы Гаусса-Зейделя – случаи, когда он нестабилен
Задавать вопрос
спросил
Изменено 4 года, 2 месяца назад
Просмотрено 890 раз
$\begingroup$
У меня есть решатель линейных систем Гаусса-Зейделя, который всегда мог решать многие виды линейных систем. Однако он борется при решении этой очень простой системы: $$ 22,7986\,x_1 + 4981,2903 \,x_2 = 25,48488 \\ 10047,4775\,x_1 -4981,2903\,x_2 = 6,05489 $$
Мое первоначальное предположение равно $(0.01,0.01)$, но я попытался изменить его на другие значения, но они все еще не сходятся.
По сути, новое предположение, которое алгоритм последовательно вычисляет на каждой итерации, становится больше, как и ошибка.
Я знаю, что существует решение для вышеуказанной системы, потому что я решил ее, используя функции Excel для инвертирования матриц, а затем использовать матричное умножение. В частности, решение системы равно $(0,003131967,0,005101786)$.
Будем рады любой помощи.
- линейная алгебра
- матрицы
- матричные уравнения
- матричное разложение
$\endgroup$
2
$\begingroup$
-4981.2903×2+10047,4775×1 = 6,05489
Шаг 1: Предположим x1 = 0 в первом уравнении и расчет x2 = 0,005116
. уравнение X1 = 0,003136
Шаг 3: Приняв X1 = 0,003136, вычислить X2 из первого уравнения X2 = 0,0051018
Шаг 4: Принять X2 = 0,0051018, вычислить X1 из второго уравнения X1 = 0,003131
Итак, окончательный ответ: X1 = 0,003131 и x2 = 0,0051018
$\endgroup$
Зарегистрируйтесь или войдите в систему
Зарегистрируйтесь с помощью Google
Зарегистрироваться через Facebook
Зарегистрируйтесь, используя электронную почту и пароль
Опубликовать как гость
Электронная почта
Требуется, но не отображается
Опубликовать как гость
Электронная почта
Требуется, но не отображается
Нажимая «Опубликовать свой ответ», вы соглашаетесь с нашими условиями обслуживания, политикой конфиденциальности и политикой использования файлов cookie

Использование исключения Гаусса-Жордана для решения следующей системы (если возможно)
Линейная алгебра Теорема Гаусса Жордана 1
Кэтлин С.
спросил 26.09.20(i) Использование исключения Гаусса-Жордана для решения следующей системы (если возможно)
Затем найдите ранги матрицы коэффициентов и расширенной матрицы и используйте теорему 1 для проверки правильности числа решений, полученных в части (i).
Подписаться І 1
Подробнее
Отчет
2 ответа от опытных наставников
Лучший Новейшие Самый старыйАвтор: Лучшие новыеСамые старые
Дениз Г. ответил 29.09.20
Репетитор
5,0 (488)
Алгебра, Колледж Алгебра, Преалгебра, Предварительное исчисление, GED, ASVAB Репетитор
Об этом репетиторе ›
Об этом репетиторе ›
В этой задаче должен быть только один правильный ответ.
1 1 1 7
1 -1 2 7
5 1 1 11
-1 R1+R2
1 1 1 7
0 -2 1 0
5 1 1 11
-5 R1+R3
1 1 1 7
0 -2 1 0
0 -4 -4 -24
-1/2R3
9 0 – 02 9 0 – 02 7 9 0 – 12 1 2 1 00 2 2 12
R2+R3
1 1 1 7
0 -2 1 0
0 0 3 12
Теперь для решения
-3 -е уравнение
3x 3 = 12 Разделите обе части на 3
x 3 =4
По второму уравнению
-2x 2 +x 3 = 0
-2x 2 +4 = 0 Вычесть 4 из обеих сторон
-2x 1 -400 2
x 2 = 2
By the 1st equation
x 1 +x 2 +x 3 = 7
x 1 +2+4 = 7 Объединить одинаковые члены
x 1 +6 = 7 Вычесть 6 с обеих сторон
x 1 = 1
(1,2,4)
Подробнее
Отчет
Нила В. ответил 29.09.20
Репетитор
Новое в Византе
Смотрите таких репетиторов
Смотрите таких репетиторов
Способ решения уравнения с помощью калькулятора ti-84:
- 2-й затем Martix затем нажмите редактировать
- используйте коэффициент перед переменными, указанными в уравнениях, и он должен быть 3 x 4, потому что это три столбца и четыре строки: 3 x 4 [ 1 1 1 7, 1 1 2 7, 5 1 1 11]
- , затем нажмите 2nd, затем Matrix, затем нажмите Math, прокрутите до B rref(
- Click 2nd и Martix
- Нажмите на (А)
Ваш ответ должен быть: rref( [A] [1 0 0 1]
[ 0 1 0 6]
[ 0 0 1 0 ]
Первый x = 1 Второй x = 6 Третий x = 0
Подставьте ответы в уравнение:
x1 + x2 + x3 = 7 1+6+0 = 7
x1 - x2 + 2×3 = 7
5×1 + x2 + x3 = 11 5(1)+ 6+0= 11
Голосовать за 0 Понизить
Подробнее
Отчет
Все еще ищете помощи? Получите правильный ответ, быстро.
